Soluções Lista V


 

Soluções Lista V Soluções:



Solução do Problema 1  



O volume de água na piscina em função de h, a altura quando h está próximo de 5 é

\begin{displaymath}V(h)=h\cdot l\cdot \frac{1}{2}(12+12+h+\frac{16h}{6})\end{displaymath}

Como l=20 ft simplificando obtemos

\begin{displaymath}V(h)=20\cdot h\cdot \frac{1}{2}\left(\frac{144+22h}{6}\right)\end{displaymath}

isto é

\begin{displaymath}V(h)=\frac{720h+122h^2}{3}\end{displaymath}

Derivando implicitamente obtemos:

\begin{displaymath}\frac{dV}{dt}=\left(24+\frac{244h}{3}\right)\frac{dh}{dt}\end{displaymath}

Como $\frac{dV}{dt}=0.8 ft/min$ temos

\begin{displaymath}\frac{dh}{dt}=\frac{3\frac{dV}{dt}}{720+244h}\end{displaymath}

isto é

\begin{displaymath}\frac{dh}{dt}=\frac{3\frac{dV}{dt}}{720+244h}=\frac{2.4}{1940}=0.012 ft/min\end{displaymath}






 
Figure 1: Piscina
\includegraphics[width=0.80\textwidth]{pisci.eps}

Solução do Problema 2  



A variação do volume de água é dada pela fórmula

\begin{displaymath}\frac{dV}{dt}={\text{entra}}-{\text{sai}}\end{displaymath}


\begin{displaymath}\frac{dV}{dt}=tx_e(t)-10000\end{displaymath}

Por outro lado como o volume de um cone é $V=\frac{1}{3}\pi r^2 h$ e da figura sabemos que $\frac{4}{r}=\frac{6}{h}$ temos que $r=\frac{2}{3}h$ e portanto

\begin{displaymath}V=\frac{1}{3}\pi \left(\frac{2h}{3}\right)^2 h=\frac{4\pi h^3}{27}\end{displaymath}

que derivando implicitamente obtemos

\begin{displaymath}\frac{dV}{dt}=\frac{4\pi h^2 \frac{dh}{dt}}{9}=tx_e(t)-10000\end{displaymath}

logo a taxa de entrada no momento em que a altura era 200cm era

\begin{displaymath}tx_e=\left(\frac{4\pi (200)^2 20}{9}+10000\right)cm^3/min\end{displaymath}

Solução do Problema 3  



Usamos a figura e a lei dos cossenos para expressar a distância entre os dois e obter:

\begin{displaymath}d^2=A^2+B^2-A B\cos \theta=
10^4+4\cdot 10^4-2\cdot 10^4\cos \theta=10^4(5-2\cos \theta)\end{displaymath}

Derivando implicitamente obtemos

\begin{displaymath}2dd'=10^4(2{\text {\ sen }}\theta) \frac{d\theta}{dt}\end{displaymath}

e então

\begin{displaymath}d'=\frac{10^4 {\text {\ sen }}\theta}{d}\frac{d\theta}{dt}.\end{displaymath}

Mas como $s=r\theta$ temos $\frac{ds}{dt}=r\frac{d\theta}{dt}$ e como $\frac{ds}{dt}=7$ m/s temos que $\frac{d\theta}{dt}=\frac{1}{r}\frac{ds}{dt}=
7\frac{1}{r}=\frac{7}{100}$. Finalmente sabendo que a distância entre eles era 200 m podemos determinar o ângulo $\theta$ a saber:

\begin{displaymath}220^2=100^2+200^2-2\cdot 10^4\cos \theta\end{displaymath}


\begin{displaymath}\cos \theta=\frac{1}{2}\end{displaymath}

implicando que $\theta=\frac{\pi}{2}$. Portanto

\begin{displaymath}d'=\frac{10^4 {\text {\ sen }}\frac{\pi}{3}}{200}\cdot 7\cdot...
...{2}{\text {\ sen }}\frac{\pi}{3}=\frac{7}{2}\frac{\sqrt{3}}{2}.\end{displaymath}

Solução do Problema 4  



Como f(x)=1-x2 e f'(x)=-2x temos que uma das tangentes, a que passa no ponto Q=(x,1-x2) tem equação

w-1+x2=-2x(v-x).

Portanto os pontos A e C são obtidos fazendo v=0 e então w=1-x2+2x2=1+x2, isto é A=(0,1+x2) e fazendo w=0 e neste caso

\begin{displaymath}\frac{x^2-1}{-2x}=u-x\Rightarrow u=\frac{x^2+1}{2x}\end{displaymath}

isto é $B=(\frac{x^2+1}{2x},0)$. A distância entre os dois é portanto:

\begin{displaymath}d=\sqrt{\left(
\frac{x^2+1}{2x}\right)^2+(1+x^2)^2}=(1+x^2)\sqrt{1+\frac{1}{4x^2}}=\end{displaymath}


\begin{displaymath}\frac{1+x^2}{2x}\sqrt{1+4x^2}\end{displaymath}

e como o triângulo deve ser equilátero devemos ter:

\begin{displaymath}d=\frac{1+x^2}{2x}\sqrt{1+4x^2}=\frac{1+x^2}{x}\end{displaymath}

que resolvendo obtemos $\sqrt{1+4x^2}=2$ isto é 1+4x2=4 e portanto

\begin{displaymath}x=\frac{\sqrt{3}}{2}.\end{displaymath}

Segue que os pontos são:

\begin{displaymath}P=\left(-\frac{\sqrt{3}}{2},\frac{1}{4}\right),\qquad
Q=\left(\frac{\sqrt{3}}{2},\frac{1}{4}\right)\end{displaymath}

Solução do Problema 5  



Sejam yh(t) a posição do homem sobre o eixo-y no instante t e (500,ym(t)) a da mulher que se desloca sobre a vertical x=500. Como as velocidades são respectivamente vh=4 e vm=5 tem-se que

\begin{displaymath}y_h(t)=4t\qquad y_m(t)=-5(t-300).\end{displaymath}

Da figura ve-se que

d2=[yh(t)-ym(t)]2+5002

que derivando implicitamente temos

dd'=[yh-ym](yh'-ym')

Logo:

\begin{displaymath}d'=\frac{[y_h-y_m](y_h'-y_m')}{\sqrt{[y_h(t)-y_m(t)]^2+500^2}}=
\frac{(y_h'-y_m')}{\sqrt{1+\frac{500^2}{[y_h(t)-y_m(t)]^2}}}\end{displaymath}

No instante t=15 como $y_h=4\cdot 15\cdot 60=60^2$ e $y_m=-5(15-5)60=-50\cdot
60$ tem-se que:

\begin{displaymath}d'=\frac{9}{\sqrt{1+\frac{500^2}{[60^2+50.60]^2}}}\end{displaymath}

Solução do Problema 6  



Da figura temos: R2=r2+h2 e portanto $h=\sqrt{R^2-r^2}$. Como o volume de um cilindro é dado por $V=\pi r^2 h$ temos:

\begin{displaymath}V=\pi r^2 \sqrt{R^2-r^2}\end{displaymath}

Derivando obtemos

\begin{displaymath}V'(r)= 2\pi r \sqrt{R^2-r^2}+ \pi r^2 \frac{-2r}{2\sqrt{R^2-r^2}}=\end{displaymath}


\begin{displaymath}\pi r\left[ 2\sqrt{R^2-r^2}-\frac{r^2}{\sqrt{R^2-r^2}}\right]=\end{displaymath}


\begin{displaymath}\pi r \left[ \frac{2(R^2-r^2)-r^2}{\sqrt{R^2-r^2}}\right]=0\end{displaymath}

isto é

2R2-3r2=0

e portanto $r^2=\frac{2}{3}R^2$

\begin{displaymath}r=R\sqrt{\frac{2}{3}}.\end{displaymath}

Portanto o volume máximo é

\begin{displaymath}V_{{\text{m\'ax}}}=\pi \frac{2}{3}R^2 \frac{1}{\sqrt{3}}R=
\frac{2\pi}{3\sqrt{3}}R^3\end{displaymath}

Solução do Problema 7  



Da figura, se denotamos y(t) e x(t) as posições dos barcos cujas velocidades são respectivamente 20km/h e 15 km/h temos que

\begin{displaymath}y(t)=20(t-2)\qquad x(t)=15t+x_0\end{displaymath}

como x(3)=0 temos x(3)=45-x0=0 e portanto x0=-45 o que acarreta x(t)=15t-45. Logo a distância entre eles será dada por:

\begin{displaymath}d=\sqrt{y(t)^2+x(t)^2}\end{displaymath}

que derivando obtemos:

\begin{displaymath}d'=\frac{2y(t)y'(t)+2x(t)x'(t)}{2\sqrt{y(t)^2+x(t)^2}}=
\frac{y(t)y'(t)+x(t)x'(t)}{\sqrt{y(t)^2+x(t)^2}}\end{displaymath}

e igualando a zero temos:

\begin{displaymath}d'=\frac{(20(t-2))20+(15t-45)15}{\sqrt{(20(t-2))^2+(15t-45)^2}}=0\end{displaymath}

isto é 400(t-2)+225(t-3)=0 625t=800+675cuja solução é:

\begin{displaymath}t=\frac{1475}{625}=2.36\end{displaymath}

Solução do Problema 8  



Para responder a) derivamos S para obter:

\begin{displaymath}\frac{dS}{dt}=-\frac{3}{2}s^2(-({\text {\ cossec }}^2 \theta)...
...rt{3}}{2}
(-{\text {\ cossec }}\theta {\text {\ cotg }}\theta).\end{displaymath}

Para responder b) igualamos o resultado obtido a zero

\begin{displaymath}\frac{3}{2}s^2({\text {\ cossec }}^2 \theta)-3s^2\frac{\sqrt{...
...[\frac{1}{2}-\frac{\sqrt{3}}{2}{\text {\ cotg }}\theta\right]=0\end{displaymath}

donde temos

\begin{displaymath}\frac{1}{2}-\frac{\sqrt{3}}{2}{\text {\ cotg }}\theta=0\end{displaymath}

isto é ${\text {\ cotg }}\theta =\frac{1}{\sqrt{3}}$ a saber as abelhas preferem o ângulo

\begin{displaymath}\theta_0=\text{arc\;{\text {\ tg }}}\sqrt{3}.\end{displaymath}

Da trigonometria sabemos que

\begin{displaymath}{\text {\ cossec }}\theta =\frac{\sqrt{1+{\text {\ tg }}^2\theta}}{{\text {\ tg }}\theta}\end{displaymath}

e portanto

\begin{displaymath}S=6sh-\frac{3}{2}s^2 \frac{1}{\sqrt{3}}+ +3s^2\frac{\sqrt{3}}{2}
\frac{2}{\sqrt{3}}\end{displaymath}


\begin{displaymath}S=6sh +3s^2\left(1-\frac{1}{2\sqrt{3}}\right).\end{displaymath}

Solução do Problema 9  



Como y=x2 e y'=2x a reta tangente à parábola no ponto (x,x2)será:

w-x2=2x(v-x).

Como esta reta deverá conter o ponto onde está a estátua que é (100,50) devemos ter:

50-x2=2x(100-x)

isto é

x2-200x+50=0

cuja solução que nos interessa é

\begin{displaymath}x_1=\frac{200-\sqrt{39800}}{2}=0.25\end{displaymath}

e portanto o ponto sobre a estrada no qual os faróis iluminarão diretamente a estátua é

(0.25, 0.252).

Solução do Problema 10  



Vamos assumir que o quadrado tem lado x e que o círculo tem raio r. Então sabemos que $4x+2\pi r=16$ e portanto $r=(16-4x)/2\pi=(8-2x)\pi$. A área total é

\begin{displaymath}A(x)=x^2+\pi
r^2=x^2+\pi\left(\frac{8-2x}{\pi}\right)^2=\frac{1}{\pi}\left((4+\pi)x^2-32x+64\right).\end{displaymath}

Calculando a derivada obtemos:

\begin{displaymath}A'(x)=\frac{2}{\pi}((4+\pi)x-16),\end{displaymath}

e portanto o único ponto crítico ocorre em $x=16/(4+\pi)$. Como estamos tratando com uma função quadrática com coeficiente do termo quadrático positivo sabemos que este é um ponto de mínimo. Portanto o corte deverá ser feito a 4x unidades da extremidade esquerda isto é a distância de

\begin{displaymath}\frac{64}{(4+\pi)}\end{displaymath}

desta extremidade.

Solução do Problema 11  



Se o cilindro (e portanto a abóboda) tem raio r e altura h, então o volume do observatório será

\begin{displaymath}V=\pi r^2 h+\frac{2}{3}\pi r^3.\end{displaymath}

Logo

\begin{displaymath}h=\frac{V}{\pi r^2}-\frac{2r}{3}.\end{displaymath}

A área da superfície cilindrica é $2\pi rh$ e a da abóboda $ 2\pi r^2$. Portanto para minimizarmos o custo da obra devemos minimizar a função:

\begin{displaymath}C(r)=2\pi rh+2(2\pi r^2)=\end{displaymath}


\begin{displaymath}2\pi \left(\frac{V}{\pi r^2}-\frac{2r}{3}\right)+
4\pi r^2=
\frac{2V}{r }-\frac{8\pi r^2}{3}.\end{displaymath}

Derivando e derivando mais uma vez obtemos:

\begin{displaymath}C'(r)=-\frac{2V}{r^2 }+\frac{16\pi r}{3}=0\qquad \quad
C''(r)=\frac{4V}{r^3 }+\frac{16\pi}{3}.\end{displaymath}

Segue que o ponto crítico de C ocorre quando $r^3=\frac{3V}{8\pi}$ e que neste ponto a derivada segunda é negativa sendo portanto um mínimo. Logo a configuração mais econômica se dá quando

\begin{displaymath}r=\sqrt[3]{\frac{3V}{8\pi}}.\end{displaymath}


 




BACK TO CALCULUS A PAGE BACK TO MATHBIO PAGE

Aldrovando Azeredo Araujo
1999-06-12